Difference between revisions of "2022 AMC 10A Problems/Problem 24"

m (Solution By Omega Learn with Complementary Counting)
Line 4: Line 4:
 
<math>\textbf{(A) }500\qquad\textbf{(B) }625\qquad\textbf{(C) }1089\qquad\textbf{(D) }1199\qquad\textbf{(E) }1296</math>
 
<math>\textbf{(A) }500\qquad\textbf{(B) }625\qquad\textbf{(C) }1089\qquad\textbf{(D) }1199\qquad\textbf{(E) }1296</math>
  
== Solution By Omega Learn with Complementary Counting==  
+
== Video Solution By OmegaLearn using Complementary Counting ==  
  
https://www.youtube.com/watch?v=jWoxFT8hRn8&list=PLT9bNzqjDoMl3jNviYrczw7Ck_ArS54Xn&index=8
+
https://youtu.be/jWoxFT8hRn8
 +
 
 +
~ pi_is_3.14

Revision as of 04:08, 12 November 2022

Problem

How many strings of length $5$ formed from the digits $0$,$1$,$2$,$3$,$4$ are there such that for each $j\in\{1,2,3,4\}$, at least $j$ of the digits are less than $j$? (For example, $02214$ satisfies the condition because it contains at least $1$ digit less than $1$, at least $2$ digits less than $2$, at least $3$ digits less than $3$, and at least $4$ digits less than $4$. The string $23404$ does not satisfy the condition because it does not contain at least $2$ digits less than $2$.)

$\textbf{(A) }500\qquad\textbf{(B) }625\qquad\textbf{(C) }1089\qquad\textbf{(D) }1199\qquad\textbf{(E) }1296$

Video Solution By OmegaLearn using Complementary Counting

https://youtu.be/jWoxFT8hRn8

~ pi_is_3.14